跳到主要內容

發表文章

目前顯示的是 2011的文章

[隨筆] 台北經驗-捷運

台北經驗 - 捷運 現代詩 - 謝宗翰 夕陽 依戀著黃昏的彩霞 在西下之前 奮力透出 最後幾縷微光 萬紫千紅的霓虹 街景,悄悄的 隨後跟上 沾染了這座不眠城市,讓 它 看起來更顯嬌媚與瘋狂 形形色色的人們,佇足在擁擠的車廂 我斜靠在車門旁,看著他們。 想知道 它們手中 提著的,身上 背著的, 以及  心中藏著的   一整天的份量。 會有多少的感動 ?  或者多少的失望 ? 轉眼兒,催促人們到站的提示聲一再播放, 原本的熙熙攘攘,轉眼間 車廂,隨之空蕩 我瞇起眼,看著人們,最後會走向何方 ? 想問問 這麼多迷濛又錯綜的步伐, 是否還有誰能記得,各自 最初想走的夢想 ?

[線性系統] 淺談動態系統的可控制性(2)

延續前篇,考慮 $n$ 維度 $p$ 組輸入的狀態方程 \[{\bf{\dot x}} = {\bf{Ax}} + {\bf{Bu}}\]由於輸出方程與控制性無關在此我們僅討論狀態方程。 我們先回憶 系統狀態可控制的定義 ===================== Definition: (Controllability for LTI system) 我們稱狀態方程 \[ {\bf{\dot x}}\left( t \right) = {\bf{Ax}}\left( t \right) + {\bf{Bu}}\left( t \right) \] 或者一組 $\left( {{\bf{A,B}}} \right)$ 為在時刻 $t_0$ 可控制(controllable),若下列條件成立: 對任意初始狀態 ${\bf{x}}\left( 0 \right) = {\bf x}_0$ 與 終止狀態 ${\bf x}_1$,存在 一組輸入訊號 ${\bf u}(t)$ 使得可以在 有限時間 內將 ${{\bf{x}}_0}$ 送至 ${{\bf{x}}_1}$。反之我們稱此 $\left( {{\bf{A,B}}} \right)$ 不可控制 (uncontrollable)。 ====================== Comment: 注意!! 上述的可控性定義並 "無" 限制 控制力大小。 現在我們給出 控制性 有關的結果整合在下面 ========================= Theorem: Controllability equivalence statements 下列 4個 陳述完全 等價(if and only if) : 1. $n$維度的 pair $(\bf{A,B})$ 為 controllable 2. 對任意 $t>0$,$n \times n$ 的矩陣 \[{{\bf{W}}_C}(t): = \int_{\rm{0}}^t {{e^{{\bf{A}}\left( {t - \tau } \right)}}{\bf{B}}{{\bf{B}}^T}{e^{{{\bf{A}}^T}\left( {t - \tau } \right)}}} d\

[分享]從事學問的目的是甚麼?

從事學問的目的是甚麼?我完全認同本日(092011)劉校長說的下面這句話 從事學問 並不是為了得獎或發表論文 , 而是為了探索新知識、新技術及對社會有所貢獻 ,讓自己 從中得到快樂 ,才能在漫長求知過程中,有源源不斷的動力。- 清大前校長、現任蒙民偉榮譽講座教授 劉炯朗, =================================== 是的,研究人員應追求學問,探索新知識,不是為了發表論文。 是要讓自己開心有動力做研究,探究學術之美不是為了升等或得獎。 希望自己也能把這些話謹記在心 :)

[數學分析] 逐點收斂與均勻收斂(4) - Uniform boundedness and Equicontinuity

回憶對於  實數sequence 而言,我們有以下結果: ================= Theorem: Convergence of Real Numbers  1. 若 $\{p_n \}$ 為一個在 compact metric space $X$ 的實數 sequence,則存在一 subsequence $\{p_{n_i}\}$ 在 $X$ 上收斂。 2. (Bolzano-Weierstrass Theorem) 任意 $\mathbb{R}^k$ 中有界 sequences 都必有收斂 subsequence。 ================= 那麼現在我們想問,如果是 函數 sequence 是否有類似結果可以使用? Q1. 任意 收斂函數sequence 是否都有均勻收斂 sequence ? Q2. 如果我們有一組 "有界" 的 函數 sequence,那麼是否此組有界函數 sequence 仍有 收斂 subsequence? 如果有? 是甚麼樣的收斂(逐點? or 均勻?) 如果沒有? 我們該怎麼修正。 讀者可以發現我們想要 模仿 實數sequence 的 Bolzano-Weierstrass theorem 到 函數 sequence 中,故第一個問題便是甚麼叫做 "有界" 的函數sequence ? 故以下我們給出 有界函數sequence (bounded function sequences)的定義 : ======================= Definition: ( Boundedness of Sequence of Function) 令 $\{f_n \}$ 為定義在 $E \subset X$ 上函數 sequence。 我們說 $\{f_n \}$ 為在 $E$ 上逐點有界(pointwise bounded)  若下列條件成立: 存在一個有限值域函數 $\phi(x)>0$ 使得 對任意 $x\in E$ 與 對 $n=1,2,3,...$,\[ |f_n(x)|< \phi(x) \] 我們說 $\{f_n \}$ 為在 $E$ 上均勻有界(uniformly bounded)  若下列條件成立: 存

[系統理論] 連續時間週期訊號的 Fourier Series Representation (2) - Coefficients determination

延續前篇,回憶 對於 週期訊號 $x(t)$ 我們可寫下其對應的 Fourier Series Representation 如下 \[ x(t) = \sum_{k=-\infty}^{\infty} a_k e^{j k \omega_0 t} \] 其中 $\omega_0$ 為週期訊號的基本頻率 (fundamental frequency)。$a_k$ 稱為 Fourier Series 的係數。之前我們已經討論過 給定 Fourier Series 係數,我們可以重建週期 $x(t)$,現在我們專注 在 給定 週期訊號 $x(t)$,如何反求 Fourier Series 的係數。 如前所述,現在給定 平滑(smooth)有界 週期訊號 $x(t)$ 且假設其可以寫下對應的 Fourier Series Representation:  \[ x(t) = \sum_{k=-\infty}^{\infty} a_k e^{j k \omega_0 t} \] 對兩邊同乘 $e^{-j n \omega_0 t}, \; n \in \mathbb{Z}$  可得 \[x(t){e^{ - jn{\omega _0}t}} = \sum\limits_{k =  - \infty }^\infty  {{a_k}{e^{jk{\omega _0}t}}{e^{ - jn{\omega _0}t}}} \]接著在對等式兩邊同積分從$0$ 積到 $x(t)$ 的基本週期(fundamental period) $T$ $(T=2\pi/\omega_0)$,亦即 \[\begin{array}{l} \int_0^T {x(t){e^{ - jn{\omega _0}t}}} dt = \int_0^T {\sum\limits_{k =  - \infty }^\infty  {{a_k}{e^{jk{\omega _0}t}}{e^{ - jn{\omega _0}t}}} } dt\\ \begin{array}{*{20}{c}} {}&{}&{}&{} \end{array} = \sum\limits_{k =  - \infty }^\infty  {{a_k}\int_0^T {{e^{j\

[系統理論] 連續時間週期訊號的 Fourier Series Representation (1) - Periodic signal represents by linear combination of complex exponentials

在系統理論中,週期訊號是非常重要的一類訊號,我們將在這篇文章介紹 對於 週期訊號的頻域處理 : Fourier Series Representation。本質上想法就是企圖將  週期訊號 透過  Complex exponentials 展開 (或者等價 用 sin 與 cos 展開)。 Comment: 1. 上述句子提及的展開 表示 週期訊號 可以透過 complex exponential 透過線性組合 建構。 2. 儘管 Fourier Series 對"大部分" 週期訊號 (e.g., 連續週期訊號)都成立。但若欲擴展到 "任意" 週期訊號的 Fourier Series Representation 須加上額外條件保證 Fourier Sereis 收斂,此部分會在後續文章再做討論。 3. 注意到若訊號為 "非週期"訊號,則 Fourier Series 不能使用,需引入 Fourier Transform!! 關於 Fourier Transform 的議題我們會在之後再做討論。 (基本想法仍不變,只是將非週期訊號 "看成" 週期訊號 但週期為無窮大) ====================== Definition: (Continuous Time Periodic Signal) 我們稱一個訊號 $x(t)$ 為週期訊號 (periodic signal) 若下列條件成立: 對任意時間 $t>0$ 存在一正實數 $T >0$,使得 \[ x(t) = x(t + T) \]====================== 下圖為連續時間的週期訊號的一個例子 我們稱 $T_0$ 為 週期訊號 $x(t)$ 的基本週期(fundamental period) 若下列條件滿足: 取最小週期 $T_0 = T>0$ 使得 $x(t) = x(t+T)$仍然成立。 由基本週期的定義,我們可透過 $\omega = \frac{2 \pi}{T}$ 定義 基本頻率 (fundamental frequency, $\omega_0$) \[ \omega_0 := \frac{2 \pi}{T_0} \]  Ex

[線性系統] 離散時間 LTI 系統的漸進穩定度

考慮離散時間系統 \[ x(k+1) = Ax(k) + Bu(k) \]若 $A$ 為 穩定矩陣,且 $u(k) \to 0$ 則 $x(k) \to 0$ Proof: 我們要證明  $x(k) \to 0$,故給定任意 $\varepsilon>0$,要證明 存在 $M>0$ 使得 對任意 $k \ge M$,我們有 \[ |x(k)| \le \varepsilon \] 注意到該系統 $x(k+1) = Ax(k) + Bu(k)$ 的解為 \[{x(k) = {A^k}x(0) + \sum\limits_{i = 0}^{k - 1} {{A^{k - 1 - j}}Bu(j)} } \]兩邊同取 norm 並利用三角不等式 可得 \[\begin{array}{*{20}{l}} {\left| {x(k)} \right| = \left| {{A^k}x(0) + \sum\limits_{i = 0}^{k - 1} {{A^{k - 1 - j}}Bu(j)} } \right|}\\ {\begin{array}{*{20}{c}} {}&{}&{}&{} \end{array} \le \left| {{A^k}} \right|\left| {x(0)} \right| + \sum\limits_{i = 0}^{k - 1} {\left| {{A^{k - 1 - j}}} \right|\left| B \right|} \left| {u(j)} \right|} \end{array}\ \ \ \ \ (*) \]回憶 Horn 與 Johnson (1985) 的結果: ================== FACT: \[ |A^k| \le c \lambda^k, \; c>0 \;\; \max_i |eig_i(A)| < \lambda <1 \]================== 故 \[\left| {x(k)} \right| \le c{\lambda ^k}\left| {x(0)} \right| + c\left| B \right|\sum\limits_{j = 0}^{\infty}

[Win8] 系統插斷異常占用CPU資源可能的解決方法

前陣子偶然發現個人 Sony Vaio 筆電 (Windows 8 作業系統) 的 "系統插斷" 程式 經常呈現異常性占用CPU資源 10~30% 左右,此類問題多半是硬體相衝所導致。下圖為 "系統插斷" 正常CPU使用情況圖 經過查詢之後發現 主因是筆電內建顯卡(Intel HD Graphics 3000) 與 獨立顯卡 (AMD Radeon 6700M) 硬體相衝問題。(多半是 Intel 顯卡有相衝問題,需要更新驅動) 解決方法很簡單,如果有發現異常 系統插斷占用,可以前往 Intel  與 AMD 官方網站 更新顯卡驅動到最新版本便可解決。

[線性系統] 淺談動態系統的可控制性(1)

一般而言控制理論中有三大重要性質 可控制性 (controllability) 可觀測性 (observability) 穩定性 (stability) 這次主要是介紹 動態系統 的 可控制性(Controllability)。 考慮 $n$ 個狀態 且 $p$ 組輸入的狀態方程 \[{\bf{\dot x}}\left( t \right) = {\bf{A}}\left( t \right){\bf{x}}\left( t \right) + {\bf{B}}\left( t \right){\bf{u}}\left( t \right) \]其中 ${\bf A}(t)$ 為 $n \times n$ 時變矩陣,${\bf B}(t)$ 為 $n \times p$ 時變矩陣。 控制性的基本想法如下: 若系統某狀態 $\bf x$ 可以透過某對應的控制力 $\bf u$ 來影響 (在有限時間 $t$ 中從任意狀態 $x_0$ 被移動到指定狀態 $x(t)$),則我們稱此狀態為可控制。 由於輸出方程與系統控制性無關,我們這邊只考慮狀態方程。 以下先給出對於線性非時變系統 其 系統可控制性的定義 ===================== Definition: (Controllability for LTI system) 我們稱狀態方程 \[ {\bf{\dot x}}\left( t \right) = {\bf{Ax}}\left( t \right) + {\bf{Bu}}\left( t \right) \] 或者一組 $\left( {{\bf{A,B}}} \right)$ 為在時刻 $t_0$ 可控制(controllable),若下列條件成立: 對任意初始狀態 ${\bf{x}}\left( t_0 \right) = {\bf x}_0$ 與 終止狀態 ${\bf x}_1$,存在時刻 $t_1 \ge t_0$ 與 一組輸入訊號 ${\bf u}(t)$ 使得可以在有限時間 $t_1$ 內將 ${{\bf{x}}_0}$ 送至 ${{\bf{x}}_1}$。反之我們稱此 $\left( {{\bf{A,B}}} \right)$ 不可控制 (uncontrollable)。 =====

[線性系統] 漸進穩定度 與 Lyapunov Theorem

這次要介紹如何 透過 Lyapunov Theorem 來檢驗線性系統 ${\bf{\dot x}} = {\bf{Ax}}$ 的漸進穩定度 (Asymptotic Stability)。關於非線性系統的漸進穩定度讀者可參考下列兩篇文章:  [系統理論] 離散時間系統的穩定度理論 (0) - 先備概念 [系統理論] 離散時間系統的穩定度理論 (1) - Lyapunov Stability Theory 現在回憶我們先前提過控制系統的兩種絕對穩定度:BIBO穩定 與 漸進穩定度。 概念上 BIBO穩定為插上電源看看系統會不會壞掉,漸進穩定則是測試拔掉電源看看系統會不會停止。 Lyapunov Energy Ideas 一般而言,Lyapunov 觀點是透過能量的角度看系統穩定度。也就是說考慮系統狀態 ${\bf{x}}\left( t \right) $,那麼  \[ {\bf{x}}\left( t \right) \to 0 \Leftrightarrow {{\bf{x}}^T}\left( t \right){\bf{x}}\left( t \right) \to 0 \] 注意到上述 ${{\bf{x}}^T}\left( t \right){\bf{x}}\left( t \right)$ 可看成能量。那麼為了達成上式,我們可以透過 能量對時間的變化率 (系統狀態能量對時間微分) 若為負值,則表示能量在逐漸溢散(decaying energy),亦即可透過 \[ \frac{d}{dt} {{\bf{x}}^T}\left( t \right){\bf{x}}\left( t \right) <0 \] 達成 ${\bf{x}}\left( t \right) \to 0 \Leftrightarrow {{\bf{x}}^T}\left( t \right){\bf{x}}\left( t \right) \to 0$ 注意:這邊我們說 ${\bf{A}}$ 矩陣為穩定若下面條件成立: 對 ${\bf{A}}$ 的所有 eigenvalue 有負實部。 現在我們看一個例子來展示 Lyapunov Energy Idea, Example 考慮 \[{\bf{\dot x}}\left( t \ri

[線性系統] LTI 系統的輸入輸出 BIBO 穩定度

這次要介紹線性系統的穩定度。一般而言在設計控制系統的時候第一步就是要檢驗系統是否穩定,如果不穩定則往往導致系統損毀。不可不慎。 一般而言控制系統穩定度可區分兩類 絕對穩定度: 指系統 是否穩定 的指標:一般而言有 BIBO 穩定 與 漸進穩定。 相對穩定度: 指系統 穩定 程度 的指標: 一般而言由 pole location,Phase Margin, Gain Margin 決定 而一般穩定度的判別方法也有兩種 Routh-Hurwitz criterion 只適用於線性系統,有興趣讀者可自行參閱任何一本自動控制教科書都會有詳細介紹。 Lyapunov energy approach 對線性/非線性系統皆適用。 Comment: 給不關心理論的讀者:事實上,在實用面上,大多時候我們可以直接使用 MATLAB 等套裝軟體直接求解 eigenvalue 並且判斷是否落在 s-plane 的左半面即可 (如果落在左半面不含虛軸,我們稱此系統 "穩定" )。 考慮一個 SISO LTI 系統描述如下: \[ y(t) = \int_0^t g(t-\tau)u(\tau)d \tau = \int_0^t g(\tau) u(t- \tau)d\tau \]其中 $g(t)$ 為系統脈衝響應(impulse response) 現在我們給出下面定義 ==================== Definition: Bounded function 一個輸入函數 $u(t)$ 稱作有界 (bounded) 如果下列條件成立: 若存在一個夠大的常數 $u_M$ 使得 \[ |u(t)| \le u_M < \infty, \; \forall t \ge 0 \]==================== 有了有界函數的定義,我們可以定義何謂 BIBO 穩定 ==================== Definition: BIBO stability 一個系統被稱為 BIBO stable (Bounded input bounded output stable) 若下列條件成立: 對任意有界輸入,系統都產生有界輸出。則此系統為 BIBO 穩定。 ======

[線性系統] 轉移函數的部分分式展開 與其對應的 反拉式轉換

令轉移函數 $G(s) := \frac{N(s)}{D(s)}$,其中 $N(s)$ 與 $D(s)$ 分別為 $s$ 的多項式,現在考慮 $G(s)$ 分母階數 比 分子階數高,亦即 \[ deg N(s) < deg D(s) \] 我們稱此 $G(s)$ 為嚴格真分有理函數 (strictly proper rational function)。 Example 1: \[ G(s) = \frac{s-1}{(s+2)(s+5)} \] 為嚴格真分有理函數。 Example 2: \[ G(s) = \frac{(s +1)(s+10)}{(s+2)(s+5)} \] 不為嚴格真分有理函數。 若 $G(s)$ 為嚴格真分有理函數,則我們可對其做部分分式展開。現在考慮某 $G(s)$ 部分分式的寫作如下: \[\begin{array}{l} G\left( s \right) = \underbrace {\frac{A}{{s + a}}}_{{\rm{single}}\begin{array}{*{20}{c}} {} \end{array}{\rm{root}}} + \underbrace {\frac{{{B_3}}}{{{{\left( {s + b} \right)}^3}}} + \frac{{{B_2}}}{{{{\left( {s + b} \right)}^2}}} + \frac{{{B_1}}}{{{{\left( {s + b} \right)}^1}}}}_{{\rm{repetitive}}\begin{array}{*{20}{c}} {} \end{array}{\rm{roots}}}\\ \begin{array}{*{20}{c}} {}&{}&{}&{} \end{array} + \underbrace {\left( {\frac{{{C_1}}}{\omega }} \right)\frac{\omega }{{{{\left( {s + \alpha } \right)}^2} + {\omega ^2}}} + \left( {\frac{{{C_2}}}{\omega }} \right)\frac{{s + \alpha

[線性系統] 動態方程式的求解(1) - LTI state equation

延續上篇,這次我們要介紹 線性非時變系統 (Linear Time-Invariant (LTI) System) 的求解。 考慮 LTI 動態系統的 狀態空間表示: \[\left\{ {\begin{array}{*{20}{l}} {{\bf{\dot x}}\left( t \right) = {\bf{Ax}}\left( t \right) + {\bf{Bu}}\left( t \right)}\\ {{\bf{y}}\left( t \right) = {\bf{Cx}}\left( t \right) + {\bf{Du}}\left( t \right)} \end{array}} \right. \]其中 $\bf{A}(\cdot), \bf{B}(\cdot), \bf{C}(\cdot),$ 與 $\bf{E}(\cdot)$ 為 $n \times n, n \times p, q \times n,$ 與 $q \times p$  常數矩陣。 我們的目標: 求解 $\bf{x}(t)$。 在求解之前我們需要一些  exponential function ${e^{  {\bf{A}}t}}$的 FACTs: 首先回憶若令 $\bf A = a$ 亦即不再是矩陣而是一個常數 $a$,則 $e^{at}$ 具有 Taylor series 如下 \[{e^{at}}: = 1 + at + \frac{{{a^2}{t^2}}}{{2!}} + ... + \frac{{{a^n}{t^n}}}{{n!}} + ...\]故若現在讓 $a$ 變回矩陣 $\bf A$ 則我們有 \[{e^{{\bf{A}}t}}: = {\bf{I}} + {\bf{A}}t + \frac{1}{{2!}}{{\bf{A}}^2}{t^2} + ... + \frac{1}{{n!}}{{\bf{A}}^n}{t^n} + ... \]那麼下面幾個性質,讀者可以使用上述定義直接驗證。 ===================== FACT 1: Inverse property \[ {e^{  {\bf{A}}t}}{e^{ - {\bf{A}}t}} = \bf{I}. \] FACT 2: Identit

[線性系統] 動態方程式的求解(0) - Review 1st ODE, DE, & $e^{At}$

在求解動態方程是之前,我們先回顧一下基本 一階常微分方程的求解: Example 1 試求解 \[ \dot {x} = a x, \ x(t_0) = x_0 \]其中 $a$ 為常數。 Solution 利用變數分離法: \[\begin{array}{*{20}{l}} {\frac{d}{{dt}}x\left( t \right) = ax\left( t \right)}\\ { \Rightarrow \int_{{t_0}}^t {\frac{1}{{x\left( \tau  \right)}}dx\left( \tau  \right)}  = \int_{{t_0}}^t {ad\tau } }\\ { \Rightarrow \left. {\ln \left( {x\left( \tau  \right)} \right)} \right|_{{t_0}}^t = a\left( {t - {t_0}} \right)}\\ { \Rightarrow \ln \left( {x\left( t \right)} \right) - \ln \left( {x\left( {{t_0}} \right)} \right) = a\left( {t - {t_0}} \right)}\\ { \Rightarrow \ln \left( {\frac{{x\left( t \right)}}{{{x_0}}}} \right) = a\left( {t - {t_0}} \right)}\\ { \Rightarrow x\left( t \right) = {x_0}{e^{a\left( {t - {t_0}} \right)}}}. \ \ \ \ \square \end{array} \] Example 2 試求解 \[ \dot {x} = a x + bu, \ x(t_0) = x_0 \]其中 $a, b \neq 0$ 為常數。 Solution 上式為標準一階常微分方程,一般而言,微分方程的解可分為兩個部分 自由響應 (free response)又稱 零輸入響應(zero input response) ;亦即令 $u(t) = 0$ 所求得的解 外力響應 (f

[隨筆] 時間謬論 Paradox of time

我想討論一個關於時間的概念,這個概念體現了這世界一個非常有趣的觀點,假設我們正處在人生的某個時間點,你可能把這段時間當成 無關緊要 甚至 稀鬆平常 的一日,但是對某些人而言,這個時間點卻可能無比重要;相反地,當某些人覺得不過是平凡一天的時光時,你卻可能正在經歷"你認為的"相當重要的事情,而且這件事會讓你覺得這個時間點即將發生的事件,將會嚴重影響妳往後人生路途,這種"所謂的"關鍵時刻蘊含相當大的焦慮與不安。 我們來假設你今天有個非常重要的行程 (基測/學測/TOEFL/GRE/面試/結婚/ whatever..) 在這個重要的時間點前,可能會感覺要無比緊張甚至認為幾乎就像是末日來臨 這時候,也許我們可以想想路上的行人,可能正在度過日復一日的"同一天" 他們也許想著每天的例行公事,走著每天必走的回家路,哼著重複的旋律 讓我們在進一步想想在這個重要時間點的隔天,隔一周,隔幾年之後的自己 當你想到當初是多麼手足無措時,會不會啞然失笑呢? 所以,讓我們試著放寬心,勇敢走下去好嗎:)

[繪圖] 塗鴉

by Chung-Han Hsieh 工具:蠟筆

[數學分析] 逐點收斂與均勻收斂(3) - Differentiation property

如果我們手邊有一個 均勻收斂的 函數 sequence $\{f_n \}$ 且假設此數列可微,我們想知道均勻收斂是否能給我們一些關於此函數sequence 微分 $\{ f_n' \}$ 的一些關聯? 首先看個例子: Example: 令 \[ f_n(x) := \frac{\sin nx}{\sqrt{n}}, \;\; (x \in \mathbb{R}, \;n=1,2,3,...) \]試回答下列問題: 1. 函數是否逐點收斂(converges pointwise)? 2. 是否均勻收斂(converges uniformly)? 3. 此函數sequence 導數 $f_n'(x)$ 為何? 4. 此函數sequence 的導數  $\{ f_n'\}$是否逐點收斂? 5. 此函數sequence 的導數  $\{ f_n'\}$是否均勻收斂? Solution 1. 首先檢驗是否逐點收斂 給定 $x \in \mathbb{R}$,我們可知道當 $n \rightarrow \infty$ 函數 sequence $\{f_n \}$為 \[\mathop {\lim }\limits_{n \to \infty } {f_n}\left( x \right) = \mathop {\lim }\limits_{n \to \infty } \frac{{\sin nx}}{{\sqrt n }} = 0 \]亦即此 $\{f_n \}$ converges pointwise 到 $0$ 2. 現在我們檢驗其是否為均勻收斂,由均勻收斂的 sup-norm 定義,我們可檢驗其 sup-norm 看是否收斂到 $0$;亦即檢驗 \[\left\| {{f_n} - f} \right\| = \mathop {\sup_{x \in \mathbb{R}} } \left| {\frac{{\sin nx}}{{\sqrt n }} - 0} \right| = \mathop {\sup_{x \in \mathbb{R}} } \left| {\frac{{\sin nx}}{{\sqrt n }}} \right| \le \frac{1}{{\sqrt n }} \]

[數學分析] 逐點收斂與均勻收斂(2) - Series version

令 $X$ 為 metric space。現在考慮 一組 函數 sequence $\{f_n \}$ 定義在集合 $E \subset X$,我們稱 $\{f_n \}$為 uniform convergence 若下列任一條件成立 1. (Definition) 對任意 $\varepsilon >0$, 存在 $N >0$ 使得 $n > N$ 對所有的 $x \in E$ \[|f_n(x) - f(x)| < \varepsilon \]2. (Cauchy criterion) 若對任意 $\varepsilon >0$, 存在 $N >0$ 使得 對任意 $x \in E$, 我們有 \[  n,m > N \Rightarrow |f_n(x) - f_m(x)| < \varepsilon \]3. (Sup-norm version) 若 \[ \lim_{n \rightarrow \infty} \sup_{x \in E} |f_n(x) - f(x)| =\lim_{n\rightarrow \infty}||f_n - f|| =0 \] 那麼現在我們看看若是一個級數而言,我們亦可討論此級數 是否 uniform convergence。故我們先給定級數收斂的定義 ==================== Definition: (Convergence of Series of numbers ) 令級數 $ \sum_{n=1}^{\infty}a_n$, 其中 $a_n \in \mathbb{R}$ ,我們稱此級數收斂若下列條件成立: 對任意 $\varepsilon >0$,存在 $N>0$ 使得 對任意 $m > N$ \[\left| {\sum\limits_{n = 1}^\infty  {{a_n}}  - \sum\limits_{n = 1}^m {{a_n}} } \right| < \varepsilon \]亦即所謂的級數的 partial sum 收斂。 ==================== 那麼對於一組函數級數的收斂該怎麼定義呢? ==================== Definit

[衍生商品] 希臘值與動態避險 (2) - Gamma and Gamma Neutrality

延續上篇 [衍生商品] 希臘值與動態避險 (1)-Delta Hedging Example ,這次要介紹 希臘值 Gamma: $\Gamma$,此參數定義為 \[ \Gamma := \frac{\partial^2 f}{\partial S^2} \] 亦即為標的資產價格 $S$ 的二次偏導數。 注意到之前我們定義過 $\Delta := \frac{\partial f}{\partial S}$,故 $\Gamma$ 可視為選擇權 $\Delta $ 的變化 與 標的資產價格 $S$ 變化的比率。 Comment 1. 當 $\Gamma $ 很小的時候,表示 $\Delta$ 變化緩慢 (stable $\Delta$) (亦即對標的資產價格變動不敏感),故此時對於 $\Delta$-Hedging 所需的 Rebalance 不需太過頻繁。但是若 $\Gamma$ 很大的時候,表示 $\Delta$ 變化劇烈,亦即對標的資產價格變動非常敏感,故此時 $\Delta$-Hedging 需要頻繁的做 Rebalance 來確保 Delta-Neutral ($\Delta =0$)。 2. 如果考慮的是一個 選擇權交易組合的 $\Gamma$,則其定義為 \[ \Gamma := \frac{\partial^2 \Pi}{\partial S^2} \] 其中 $\Pi$ 為選擇權交易投資組合的價格。 假定且我們假設 標的資產的波動度為 Constant,則投資組合的價格為資產價格 $S$ 與 時間 $t$ 的函數,亦即我們可對 $\Delta \Pi$ 做泰勒展開求資產價格的變化 \[ \small{\Delta \Pi  = \underbrace {\frac{{\partial \Pi }}{{\partial S}}}_\Delta \Delta S + \underbrace {\frac{{\partial \Pi }}{{\partial t}}}_\Theta \Delta t + \frac{1}{2}\underbrace {\frac{{{\partial ^2}\Pi }}{{\partial {S^2}}}}_\Gamma \Delta {S^2} + \frac{1}{2}\frac{